16 Rosh Review 2018

Ace your homework & exams now with Quizwiz!

Paroxysmal junctional tachycardia is usually in what range of heart rates?

150 to 250 beats per minute.

Pediatric dosing for intravenous fluid boluses in hypotensive pediatric patients

20 milliliters per kilogram

What is a rebound headache?

A chronic or nearly daily headache associated with frequent use of medication for acute head pain. On a slightly related note, cluster headaches are sudden-onset unilateral repetitive brief headaches with injection, lacrimation, or rhinorrhea. Treat with high-flow oxygen. Home treatment with calcium channel blockers.

Question: What medication is recommended for the treatment of atrial dysrhythmias associated with hyperthyroidism?

Answer: Propranolol. Rapid Review Hyperthyroidism Patient will be complaining of heat intolerance, palpitations, weight loss, tachycardia, and anxiety PE will show hyperreflexia Labs will show low TSH and high free T4 Most commonly caused by Graves disease (autoimmune against TSH receptor) Treatment is methimazole or PTU Comments: Propylthiouracil (PTU) P for pregnant

Question: How many joules are recommended for defibrillation of pulseless electrical activity?

Answer: Pulseless electrical activity (PEA) is not a shockable rhythm; defibrillation is not indicated. Rapid Review Ventricular Fibrillation Most common cause: ischemic heart disease ECG: irregular chaotic pattern without P waves or QRS complexes Immediate defibrillation

Question: What is the most important condition to exclude before confirming a diagnosis of toxic synovitis?

Answer: Septic arthritis.

What can result from circumferential eschar from a full-thickness burn?

Compartment syndrome.

A patient with bacterial conjunctivitis who wears contact lenses daily needs additional antibiotic coverage for which pathogen?

Contact lenses are at an increased risk of infection with Pseudomonas aeruginosa. Preauricular adenopathy usually suggestive of viral conjunctivitis.

An obese 11-year-old Cherokee boy presents with symmetrical, hyperpigmented, velvety plaques in the intertriginous areas of the axilla, groin, and posterior neck. There are a few acrochordons scattered in the plaques. The plaques are painless, and do not itch. What is the most likely diagnosis? Acanthosis nigricans. Addison's disease. Dowling-Degos disease. Pellagra.

Acanthosis nigricans. Acanthosis nigricans is a thickening and hyperpigmentation of the intertriginous regions of the skin. Acanthosis nigricans is more common in those with Native American, African-American, or Hispanic heritages than in Caucasians. The condition is commonly associated with other conditions such as insulin resistance and obesity, but it has also been associated with certain genetic syndromes, malignancies and drug reactions. Since acanthosis nigricans is often an asymptomatic disorder, cosmetic concerns are typically the primary indications for treatment. Treatment of the underlying cause is the preferred method of management. Otherwise, topical retinoids are the preferred treatment of the plaques because of their keratinolytic effects. Cutaneous hyperpigmentation is present in nearly all patients with Addison's disease (B) because the cortisol deficiency leads to an overproduction of melanin. The hyperpigmentation is generalized across the entire body with Addison's, but it is most conspicuous in areas of high friction or pressure, such as the elbows, knees, waist and knuckles. Most patients with Addison's complain of fatigue, weakness, anorexia and weight loss. These symptoms and the generalized nature of the rash are not consistent with our patient. Dowling-Degos disease (C) is a rare, genetic disorder that presents with reticulated hyperpigmentation and has a predilection for flexural areas. This reticulated pattern is not consistent with the plaques seen on the presented patient. Lesions also typically appear in early adulthood, not at 11-years-old. Pellagra (D) is also known as niacin (vitamin B3) deficiency. Pellagra presents with a symmetric hyperpigmented rash, in the exposed areas of skin. The rash is a photosensitive dermatitis and thus only appears where there is sun exposure. The patient may also have a red tongue, diarrhea, vomiting, insomnia, anxiety, disorientation, delusions, dementia, and encephalopathy. Niacin deficiency is very rare in the Western world but may be seen with alcoholism, anorexia nervosa, gastric bypass or malabsorptive diseases.

Question: How much calcium and vitamin D should the female athlete add to their diets?

Answer: 1200-1500 mg of calcium and 400 IU of vitamin D. Rapid Review Bulimia Binge eating Persistent overconcern with body shape and weight Submandibular gland enlargement Calluses over dorsal aspect of fingers, dental enamel erosion Cognitive behavioral therapy

Question: What is the recommended blood pressure goal in patients with hypertension?

Answer: 139/89 mm Hg or less.

Question: What percentage of ovarian serous cystadenocarcinomas are bilateral at presentation?

Answer: 30%. Rapid Review Ovarian Cancer Most lethal gynecologic malignancy 55 to 65 years Risk factors: family history, age, ↑ estrogen exposure Routine screening not indicated Postmenopausal woman with new-onset ascites Serous cystadenocarcinoma: most common, often bilateral CA-125: useful in monitoring disease course Ultrasound

Question: What percentage of children diagnosed with ADHD have more than one psychiatric diagnosis?

Answer: 65%. Rapid Review Attention-Deficit/Hyperactivity Disorder (ADHD) Most common childhood behavioral disorder Inattention + impulsivity + hyperactivity Dx criteria: Sx present in 2 areas of interaction Sx must present before 12 years Sx > 6 months Sx maladaptive/inappropriate for child's developmental stage Rx: medication (most effective), behavioral modification, environmental intervention

Question: What is the Jarisch-Herxheimer reaction?

Answer: A febrile reaction to antigens that are liberated when spirochetal bacteria (classically syphillis) are destroyed by antibiotic therapy. Rapid Review Sulfa Allergy Popular FACTSSS: Probenecid, Furosemide, Acetazolamide, Celecoxib, Thiazides, Sulfonamides, Sulfasalazine, Sulfonylureas

Question: What are some risk factors are associated with developing colon cancer?

Answer: Age >50, African-American race, history of colorectal polyps, inflammatory intestinal conditions, inherited syndromes, family history in an immediate family member. Rapid Review Colorectal Cancer Second leading cause of death Third most common cancer in men and women Adenocarcinoma Risk factors: age, IBD, adenomatous polyps, FAP, HNPCC Rectosigmoid > ascending > descending Left-sided cancer: tends to obstruct Right-sided cancer: tends to bleed Iron deficiency anemia Colonoscopy CEA

Question: What is rheumatic heart disease (also known as rheumatic fever)?

Answer: An antibody-cross reactivity inflammatory disease which follows Streptococcus pyogenes infection and causes heart (myocarditis), joint, skin and brain inflammation, most commonly occurring in 6-15 year olds. Rapid Review Tricuspid Stenosis Most common cause: rheumatic heart disease almost always occurs with mitral stenosis (MS) Murmur: Diastolic murmur along left sternal border Louder than MS during inspiration JVP: giant "a" waves

Question: List some medications whose side effects can cause symptoms of orthostasis?

Answer: Antiparkinsonian drugs, antiadrenergics, anticholinergics, antidepressants, antiarrhythmics, antipsychotics, diuretics, narcotics and sedatives. Rapid Review Orthostatic Hypotension Decrease in systolic blood pressure ≥ 20 Decrease in diastolic blood pressure ≥ 10 Inadequate physiologic response to postural changes

Question: Does alcohol help an essential tremor or is this a myth?

Answer: At least 50% of patients note improvement or complete amelioration of tremor following the ingestion of a small amount of alcohol. Rapid Review Essential Tremor Patient with a history of a family member with similar symptoms Complaining of hand tremor that is exacerbated by action and improved after alcohol consumption Most commonly caused by autosomal dominant Treatment is propanolol

Question: What is the most useful test to confirm a diagnosis of sarcoidosis?

Answer: Biopsy of the affected organ. Rapid Review Sarcoidosis African-Americans, females Primary target organ: lungs Parotid enlargement Hypercalcemia CXR: bilateral hilar adenopathy Biopsy: noncaseating granulomas Steroids

Question: Which symptoms are classically associated with metastatic prostate cancer?

Answer: Bone pain, back pain and several neurologic symptoms associated with spinal cord compression. Rapid Review Prostate Cancer RFs: age (most important), African-Americans, family hx Asymptomatic until advanced Obstructive uropathy Back pain: metastases to lumbar spine (↑ ALP) PSA > 10 ng/mL Dx: needle core biopsy

Question: What is the mechanism of action of clopidogrel?

Answer: Clopidogrel binds to the platelet adenosine diphosphate (ADP) receptor to irreversibly inhibit activation and aggregation for the life of the platelet. Rapid Review Acute Coronary Syndrome: Management Aspirin: ↓ mortality, ↓ infarct size, ↓ reinfarction rate Clopridogrel: patients with aspirin allergy Heparin: ↓ DVT, ↓ reinfarction, ↓ stroke, ↓ LV thrombus, ↓ reocclusion Nitroglycerin: Coronary artery dilation/vascular smooth muscle relaxation → ↓preload/afterload → ↓ myocardial O2 demand Contraindications: sildenafil use within 24 hrs, RV infarction ß-blockers: ↓ Myocardial O2 demand, ↓ ventricular fibrillation IV indications: tachydysrhythmias, intractable HTN Morphine: ↓ Preload/afterload, ↓ sympathetic activity No mortality benefit Glycoprotein IIb/IIIa inhibitors: benefit in patients undergoing PCI PCI: Preferred over thrombolytics in all STEMI patients PCI center: <90 minutes contact to device time Non-PCI center: transfer to PCI center if contact to device time can be <120 minutes Non-PCI center: thrombolytics if contact to device time to be >120 minutes Thrombolytics: begin within 30 minutes of ED arrival if selected

Question: List some risk factors for an anorectal abscess?

Answer: Crohn's disease, diabetes mellitus and chronic corticosteroid use. Rapid Review Perirectal Abscesses Ischiorectal, intersphincteric, supralevator, postanal Dull achy rectal pain CT may be obtained OR drainage Complication: fistula

Question: What percentage of sudden unexpected death of infancy (SUDI) deaths is later discovered to have died of child abuse?

Answer: Fewer than 5% of cases. Rapid Review Sudden unexpected death of infancy (SUDI) Peak incidence: 2-4 mos Risk factors: maternal smoking/drug use, prone sleeping position Recommendations: supine sleeping, pacifiers, breastfeeding

Question: What structure is often seen on peripheral smear of the G6PD patient?

Answer: Heinz bodies. Rapid Review G6PD Deficiency X-linked recessive Asymptomatic until exposed to oxidative stress Antimalarials, sulfonamides, nitrofurantoin, methylene blue, fava beans, vitamin K Heinz bodies Hemolytic anemia

Question: What systems need to be monitored for side effects from carbamazepine?

Answer: Hematologic (CBC) and hepatic (liver function tests). Rapid Review Trigeminal Neuralgia Sudden unilateral paroxysms of pain in gums, cheek, chin, temporal forehead Pain in V2 and V3 distributions, not V1 Right side > left side Triggers: chewing, brushing teeth, touching face, hot/cold exposure Carbamazepine

Question: How is babesiosis identified on a peripheral smear?

Answer: Identification of intracellular organism in erythrocytes.

Question: What is seen on electron microscopy of the glomerulus in patients with poststreptococcal glomerulonephritis?

Answer: Immune deposits in the subepithelial space.

Question: Which organisms are commonly found in tubo-ovarian abscesses?

Answer: Infection is often polymicrobial and can include E. Coli, aerobic streptococci, and Bacteroides. Rapid Review Tubo-ovarian Abscess Patient will be a woman with a history of pelvic inflammatory disease (PID) Complaining of lower abdominal pain, fever, vaginal discharge PE will show unilateral adnexal tenderness Diagnosis is made by ultrasound Most commonly caused by a complication of pelvic inflammatory disease Treatment is intravenous antibiotics and surgical drainage

Question: Name a serious complication of mitral stenosis?

Answer: Long-standing mitral stenosis can lead to left atrial dilation, a prominent risk factor for atrial fibrillation and thromboembolism. Rapid Review Mitral Stenosis Most common cause: rheumatic heart disease Dyspnea on exertion, hemoptysis Opening snap, diastolic apical murmur Atrial fibrillation

Question: What is the most common condition associated with placental abruption?

Answer: Maternal hypertension, including essential hypertension, gestational hypertension and preeclampsia. Rapid Review Postpartum Hemorrhage Patient will be a woman who has just given birth PE will show > 500 cc loss of blood and an enlarged "boggy" uterus Most commonly caused by uterine atony Treatment is uterine massage, oxytocin, prostaglandins or surgery

Question: Which nerve provides innervation to the ventral surface of the thumb, index, and long finger?

Answer: Median nerve.

Question: Pellagra is caused by a deficiency in what nutrient?

Answer: Niacin. Rapid Review Acanthosis Nigricans Patient will be obese or diabetic PE will show thickened, velvety, darkly pigmented plaques on the neck or axillae Comments: Screen for diabetes in those not yet diagnosed

Question: According to the new JNC8 guideline, how much time needs to pass before any adjustments to medication are made for hypertension management?

Answer: One month. Rapid Review Hypertension: Eighth Joint National Committee (JNC 8) Recommendations PreHTN: systolic blood pressure (SBP) 120-139 mmHg or diastolic blood pressure (DBP) 80-89 mmHg Stage I HTN: SBP 140-159 mmHg or DBP 90-99 mmHg Stage II HTN: SBP >160 mmHg or DBP >100 mmHg Treatment goals: >60 years: SBP <150, DBP <90 All others: SBP <140, DBP <90 1st line rx for general population: thiazide, CCB, ACEI, or ARB 1st line rx for African Americans: CCB or thiazide Chronic kidney disease: rx should include ACEI or ARB

Question: What physical exam finding would make a patient's alopecia areata less likely to respond to corticosteroid treatment?

Answer: Patients with involvement of the facial or body hair are often less likely to show improvement when steroids are used. Rapid Review Alopecia Areata Patient with a history of an autoimmune disorder Complaining of hair loss PE will show patches of smooth, non-scarring hair loss with patches of smaller hairs termed "exclamation hairs" Most common cause is autoimmune Treatment is intralesional corticosteroids

Question: What physical exam finding is a hallmark of pericarditis?

Answer: Pericardial friction rub. Rapid Review Acute Pericarditis Idiopathic > viral (Coxsackie) Pleuritic chest pain radiating to the back Pain ↓ with leaning forward Pericardial friction rub ECG: diffuse STE, PR depression NSAIDs

Question: What is the most common sign of ITP?

Answer: Petechiae. Rapid Review Idiopathic Thrombocytopenic Purpura (ITP) Patient will be a child 2 - 6 yrs old With a history of recent viral infection Complaining of red spots on skin or easy bleeding PE will show petechiae, purpura, and gingival bleeding Labs will show platelets < 50,000 µL Most commonly caused by antiplatelet antibodies Treatment is observation, steroids, IVIG

Question: What is the treatment for hyperkalemia resulting in metabolic acidosis?

Answer: Sodium bicarbonate. Rapid Review Hyperkalemia Patient with a history of renal failure, DKA, rhabdomyolysis, tumor lysis Complaining of lethargy, weakness, paralysis PE will show bradycardia, hypotension, cardiac dysrhythmia ECG will show peaked T waves, prolonged PR, wide QRS Treatment is calcium gluconate, insulin, albuterol, kayexalate, bicarbonate

Question: A pleural effusion is most difficult to detect in which radiographic position?

Answer: Supine. Rapid Review Pleural Effusion Transudate: CHF (most common) Exudate: infection > malignancy, PE ↓ Breath sounds + dull percussion + ↓ tactile fremitus CXR: blunting of the costophrenic angle

Question: An S3 extra heart sound is heard best with which end of a stethoscope?

Answer: The bell. Rapid Review Dilated Cardiomyopathy Causes: idiopathic > viral myocarditis, Chagas disease Echo: 4 dilated chambers (ventricles > atria) Systolic pump failure → heart failure S3, S4 Peripheral embolization of mural thrombi MC dysrhythmia: atrial fibrillation ßBs, ACEIs, diuretics, digoxin, anticoagulation, AICD

Question: What are 3 consequences of PID?

Answer: Tubo-ovarian abscess, infertility, ectopic pregnancy. Rapid Review Pelvic Inflammatory Disease (PID) Patient will be a Female With a history of multiple sexual partners or unprotected sex or both Complaining of lower abdominal pain, vaginal discharge, painful sexual intercourse PE will show mucopurulent cervical discharge, cervical motion tenderness ("Chandelier sign") Most commonly caused by Chlaymidia Treatment is ceftriaxone + doxycycline Comments: Fitz-Hugh-Curtis syndrome: perihepatitis + PID

Question: How soon upon arrival should internationally adopted children be evaluated by a healthcare professional?

Answer: Two weeks.

Question: What is the most common ECG finding in hypokalemia?

Answer: U waves. Rapid Review Hyperkalemia Patient with a history of renal failure, DKA, rhabdomyolysis, tumor lysis Complaining of lethargy, weakness, paralysis PE will show bradycardia, hypotension, cardiac dysrhythmia ECG will show peaked T waves, prolonged PR, wide QRS Treatment is calcium gluconate, insulin, albuterol, kayexalate, bicarbonate

Question: In what situations is a finger sweep appropriate for a choking child?

Answer: When the child has a complete airway obstruction as manifest by inability to speak or cough. Rapid Review Foreign Body Aspiration Patient will be a child Complaining of a sudden episode of coughing, wheezing, or stridor Comments: most common is the right main bronchus

What is the most common cause of vertigo?

Benign (paroxysmal) positional vertigo. On an unrelated note, acoustic neuroma (cranial nerve 8) causes gradual hearing loss, tinnitus, and ataxia. MRI with contrast.

Lady, 67. Altered mental status. Potassium 6.4. ECG shows peaked T-waves. Treatment? Albuterol. Calcium gluconate. Furosemide. Sodium polystyrene sulfonate.

Calcium gluconate. Hyperkalemia is defined as a serum potassium concentration greater than 5.5 mEq/L in adults. As the potassium concentration increases, the patient is at increased risk of a lethal dysrhythmia. In patients with severe hyperkalemia, treatment focuses on immediate stabilization of the myocardial cell membrane. Although calcium gluconate has no effect on the plasma potassium level, it should be given when there are electrophysiologic changes in the setting of hyperkalemia in order to stabilize the cardiac membrane and reduce the risk of dysrhythmias. Calcium has no effect on the serum level of potassium. For that reason, administration of calcium should be accompanied by the use of other therapies that actually help lower serum potassium levels. Definitive therapy is hemodialysis in patients with renal failure or when pharmacologic therapy is not sufficient. Therapies that translocate potassium from the serum to the intracellular space should be instituted next, as they can quickly, albeit temporarily, lower the plasma concentration of potassium. Albuterol (A) is an adrenergic agonist that has an additive effect with insulin and glucose, which may in turn help shift potassium into the intracellular space. If the patient has only a moderate elevation in potassium and no electrocardiographic abnormalities, excretion can be increased by using a diuretic such as furosemide (C). Total body potassium can be lowered with sodium polystyrene sulfonate (D), but this takes longer to affect the plasma potassium level than translocation methods and Multiple doses are usually necessary.

Which is true regarding allergic drug reactions? Celecoxib can cause a reaction in patients with a sulfonamide allergy. Cross-reactivity between penicillins and first generation cephalosporins is 50%. Patients with a history of anaphylaxis to penicillin can never be given penicillin. The onset of serum sickness generally occurs within one to two days.

Celecoxib can cause a reaction in patients with a sulfonamide allergy. Celecoxib contains a sulfonamide moiety and may cross-react if administered to patients with a sulfa drug allergy. The true cross-reactivity rate is unclear. Other medications that contain a sulfonamide moiety include most loop and thiazide diuretics and sulfonylureas such as glipizide. It is best to avoid these medications in patients with severe sulfa allergy. The rate of cross-reactivity of penicillins and first generation cephalosporins is approximately 1%-7%, not 50% (B). Third and fourth generation cephalosporins have a much lower cross reactivity than first generation, closer to 1%. Many patients with a penicillin allergy can tolerate second- and third-generation cephalosporins without issue. However, patients with life-threatening reactions to penicillin should not receive cephalosporins in the ED unless absolutely necessary. Patients with a history of penicillin anaphylaxis should not be given penicillin or cephalosporins, however, there are a few selected indications where desensitization (C) should be performed. An example is the pregnant patient with syphilis and a penicillin allergy. Careful desensitization should occur in the ICU with increasing doses of the medication. Serum sickness (D) is an immune-complex mediated reaction characterized by malaise, joint pain, urticaria, fever, adenopathy, and hepatosplenomegaly. Symptoms usually begin one to two weeks after drug exposure and may take several weeks to resolve. Treatment is generally supportive with corticosteroids administered for more severe cases.

Man, 65. Chest pain. Want to give aspirin, but angioedema previously. What to do? Clopidogrel. Dipyridamole. Lower dose, 81 milligrams aspirin and observe closely for angioedema. Pretreat with corticosteroids and antihistamines and administer full-dose aspirin.

Clopidogrel. A 162-325 mg dose of aspirin taken early in the course of myocardial infarction (MI) has been shown to produce a 23% reduction in 30-day mortality. Patients with an aspirin allergy are at risk for losing this benefit. The use of clopidogrel was shown in the CAPRIE trial to be a sufficient antiplatelet inhibitor when compared to aspirin. Therefore, in patients with true aspirin allergies, clopidogrel should be substituted for aspirin. Dipyridamole (B) is used to inhibit thrombus formation but has not been shown to reduce mortality in acute coronary syndrome. It is commonly used in combination with aspirin for the secondary prevention of stroke and TIA. With a true aspirin allergy, all doses of aspirin (C & D) are contraindicated.

A 57-year-old man presents complaining of a change in stools over the last 4 months. His stools are intermittently bloody, and he has experienced a decline in his appetite associated with a 15-pound unintentional loss of weight. Which of the following is the most likely explanation for his symptoms? Adenocarcinoma of the colon Gastrointestinal carcinoid tumor Primary colorectal lymphoma Squamous cell carcinoma of the colon

Correct Answer ( A ) Explanation: Adenocarcinoma of the colon is the most common type of colorectal cancer and the second leading cause of death due to cancer in the United States. Symptoms may include abdominal pain, change in bowel habits, weight loss, loss of appetite and hematochezia. Diagnosis is usually accomplished by colonoscopy with biopsy, though CT colonography or barium enema may be considered, as well. CT or MRI may be used for help with staging. Other laboratory findings may include iron-deficiency anemia, elevated liver transaminase levels, and an elevated carcinoembryonic antigen level. Treatment includes resection of the cancer with regional lymph node dissection, as well as chemotherapy and radiation for patients with more advanced disease. Gastrointestinal carcinoid tumor (B) is the most frequently occurring neuroendocrine tumor of the gastrointestinal tract. It is uncommon, though the true incidence is unknown, as carcinoid tumors often cause no symptoms and are found incidentally at the time of surgery performed for another primary malignancy or other reason. Resection of the tumor is often sufficient for treatment and the prognosis is favorable for tumors which have not spread. The gastrointestinal tract is the most common site for the occurrence of carcinoid tumors. Primary colorectal lymphoma (C) is a rare condition and most commonly occurs in the cecum. It is more common in older males, and presents with abdominal pain and weight loss. It is often diagnosed at a later stage due to the non-specific nature of its symptoms. The GI tract is the most common site for extra-nodal occurrence of lymphoma. Squamous cell carcinoma (D) of the colon is a rare form of colonic cancer, with little written about it in the medical literature due to its low incidence. Symptoms are similar to those of patients diagnosed with adenocarcinoma, though it is often diagnosed at later stages and thus, a poor prognosis for those with this condition.

Which of the following ECG findings is most characteristic of a premature junctional contraction? Inverted P' wave following the QRS Peaked T wave Premature P' wave Widened QRS > 120 msec

Correct Answer ( A ) Explanation: An inverted P' (P-prime) wave following the QRS is most characteristic of a premature junctional contraction. A premature junctional contraction is the result of an irritable automaticity focus in the AV junction which fires a premature stimulus that is conducted to, and depolarizes, the ventricles and often the atria as well. The premature junctional contraction captures the atria in a retrograde fashion and the ventricles in an antegrade fashion. Since atrial and ventricular depolarizations are moving in opposite directions from the junctional focus, the premature P' wave is inverted or opposite the QRS. Bottom up depolarizations record as an inverted P' wave in ECG leads with an upright QRS. The retrograde P' wave may appear before, during, or after the QRS complex. A premature P' wave (C) is seen with premature atrial contractions which record as a P' on ECG. An atrial focus is the origin of this premature atrial contraction, not the SA node, so the stimulus produces a premature and unusually shaped P' wave that looks different than the normal sinus generated P wave. A widened QRS > 120 msecs (D) can be induced by intrinsic or extrinsic factors, including but not limited to, a premature ventricular beat, or pre-excitation of the ventricles via a bypass tract as in Wolff-Parkinson-White and intraventricular conduction delays. Intraventricular conduction delays refer to abnormalities in the intraventricular propagation of supraventricular impulses. These abnormalities of propagation give rise to changes in the shape and duration of the QRS complex. Examples include left and right bundle branch block patterns and their variants. Peaked T waves (B) can be seen with hyperkalemia and are not associated with premature junctional contractions.

A 52-year-old man presents for a health maintenance appointment. Although he has no family history of prostate cancer, he wants to be screened for it. He is otherwise healthy. According to the American Cancer Society (ACS) screening recommendations, you would offer which of the following services? Digital rectal examination and serum prostate-specific antigen level Digital rectal examination only No screening tests are recommended for this age group Transrectal, ultrasound-guided biopsy

Correct Answer ( A ) Explanation: Cancer of the prostate represents the most common cancer in men, and the 2nd most common cause of cancer death in men. Lifetime risk is 16%. Risk factors include age, African American race and a positive family history. Symptoms include the obstructive symptoms similar to those of benign prostatic hyperplasia (nocturia, retention, decreased stream and hesitancy), as well as irritating symptoms similar to those of prostatitis (urgency, frequency and dysuria). Staging involves transrectal ultrasound guided biopsy, imaging and assigning a Gleason grade. Since mortality benefit from screening is controversial, and 15% of men with a normal prostate-specific antigen (PSA) level and digital rectal exam (DRE) have biopsy-proven disease, screening recommendations are varied. The American Cancer Society (ACS) recommends employing shared decision making regarding screening for men >50 years old (>45 years old if high risk) with life expectancy >10 years with a serum prostate-specific antigen level and possibly a digital rectal exam. In contrast, the US Preventive Services Task Force recommends against PSA screening. Treatment options include radiation therapy, androgen deprivation therapy, chemotherapy and radical prostatectomy. Median survival of metastatic disease is 24 to 30 months. The ACS recommends PSA and an optional DRE (A). This patient matches criteria for DRE and PSA screening (C). Transrectal, ultrasound-guided biopsy (D) is used to stage disease once it is diagnosed. It also plays a diagnostic role, however, it would not be offered as a screening tool in a patient without urinary symptoms.

A 41-year-old previously healthy woman presents to the ED with gingival bleeding and epistaxis over the past 4 days. She takes no medications and has no recent travel. On exam, you note scattered petechiae, but otherwise it is normal. Her hemoglobin is 12.5 g/dL, INR 1.1, PTT 25, and platelets 14,000/µL. A peripheral blood smear shows few large, well-granulated platelets. Which of the following is the most appropriate initial treatment? Corticosteroids and immunoglobulin Observation Platelet transfusion Splenectomy

Correct Answer ( A ) Explanation: Idiopathic thrombocytopenic purpura (ITP) is an acquired autoimmune disease that results in rapid destruction of platelets. It is characterized by thrombocytopenia, the presence of purpura or petechiae, normal bone marrow, and no other identifiable cause for the thrombocytopenia. It is more common in children than in adults. The physical exam may reveal petechiae, epistaxis, gingival bleeding, and menorrhagia. The presence of lymphadenopathy, hepatosplenomegaly, pallor, or hyperbilirubinemia should suggest an alternative diagnosis such as leukemia, lymphoma, lupus, mono, or hemolytic anemia. Management is predicated primarily on the severity of thrombocytopenia and bleeding. Drugs that interfere with platelet function are discontinued. In general, asymptomatic patients with platelet counts >50,000/µL require no treatment. Prednisone is the initial therapy for adult patients with platelets < 50,000/µL who are not actively bleeding but have other signs (ecchymoses, petechiae, etc). When the platelet count is < 20,000/µL, IV methylprednisolone is preferred. Intravenous (IV) immunoglobulin is co-administered for patients with counts < 20,000-30,000/µL and active bleeding. Only if bleeding is significant after administration of methylprednisolone and immunoglobulin should platelets be transfused (C). Splenectomy (D) is rarely performed in patients with ITP but should be considered when the condition is refractory to medications. Observation (B) is appropriate for patients who are asymptomatic with platelet counts > 50,000/µL.

What lab abnormality is commonly seen in sarcoidosis? Hypercalcemia Hypocalcemia Hypophosphatemia Neutropenia

Correct Answer ( A ) Explanation: Sarcoidosis has a number of lab abnormalities associated with it including hypercalcemia. Sarcoidosis is a multisystem inflammatory disease that involves non-caeseating granulomas composed of collections of T-helper cells and other inflammatory cells. It predominantly affects the lungs and intrathoracic lymph nodes but can involve a number of systems including cardiac, skin and neurologic. Chest radiography often reveals hilar adenopathy in early stages and progresses to pulmonary fibrosis. Patients may present with a host of non-specific symptoms including fever, fatigue, weight loss and polyarthritis. Other symptoms depend on end organ involvement. The granulomas secrete 1,25 vitamin D leading to hypercalcemia and hypercalciuria. Hypocalcemia (B) is not seen in sarcoidosis. Hyperphosphatemia (C), due to impaired renal phosphate excretion, not hypophosphatemia is commonly seen in this disorder. Patients often have elevated leukocyte counts (D) with a predominance of eosinophilia.

Which of the following is the most common cell type of ovarian cancer? Epithelial Germ cell Granulosa cell Stromal

Correct Answer ( A ) Explanation: The most common primary type of ovarian cancer is of epithelial cell origin (90%), followed by germ cell origin (<5%, but the most common type in women younger than 20 years of age), then stromal cell origin (<2%). The most common malignant epithelial cell ovarian tumor is a serous cystadenocarcinoma. The most common malignant germ cell ovarian tumors are dysgerminoma and teratoma. The most common malignant stromal cell ovarian tumor is an estrogen-secreting granulosa cell tumor which can lead to endometrial hyperplasia and carcinoma. Metastases are possible, and most commonly originate from the stomach, breast and endometrium. Germ cell (B), granulosa cell (C), stromal (D) type of cancers are all less common than epithelial.

A 55-year-old man on dialysis presents with the above ECG. What treatment should be immediately administered? Calcium gluconate Insulin Sodium bicarbonate Sodium polystyrene sulfonate

Correct Answer ( A ) Explanation: This patient presents with a potentially lethal dysrhythmia from hyperkalemia and should immediately be treated with intravenous calcium salts (e.g. calcium chloride or gluconate). Calcium gluconate can be used with a peripheral line while it is recommended that calcium chloride by used with a central line. Hyperkalemia is defined as any serum potassium level > 5.0 mEq/L and is the most dangerous acute electrolyte disorder. Elevated potassium levels cause cardiotoxicity by increasing the resting membrane potential of cardiac myocytes. This makes the membranes more excitable and decreases the duration required for depolarization. Hyperkalemia can cause a number of cardiac dysrhythmias including heart blocks, bradycardia, ST segment elevation and a "sine wave" pattern. Eventually, patients develop cardiac arrest from decompensation to ventricular fibrillation. In patients with hyperkalemia causing a wide QRS complex, bradydysrhythmias or heart block, intravenous calcium should be immediately administered. Calcium acts by stabilizing the cardiac membrane via restoration of the electrical gradient. The effects of calcium are relatively rapid (within 5 - 10 minutes). Insulin (B) and sodium bicarbonate (C) are treatments that shift potassium from the extracellular space to the intracellular space but take at least 15 minutes to have effect. Sodium polystyrene sulfonate (D) is a resin, which binds potassium in the gut leading to elimination. The onset of action of this drug is hours.

A 33-year-old woman presents with intermittent, intense shooting pain to the left side of the face. She states that the pain begins near her ear and radiates to her chin. The pain is often brought on by chewing and brushing her teeth. Which of the following managements is most likely indicated? Carbamazepine Dental X-rays Mandible CT scan Prednisone

Correct Answer ( A ) Explanation: This patient presents with symptoms consistent with trigeminal neuralgia and should be treated with carbamazepine. Trigeminal neuralgia is a syndrome characterized by sudden paroxysms of lancinating pain in one or more of the trigeminal nerve distributions. The syndrome manifests with unilateral facial pain triggered by chewing, brushing the teeth or touching the affected areas of the face. Hot and cold temperature exposure can provoke symptoms as well. The maxillary (V2) and mandibular (V3) branches of the trigeminal nerve are most commonly affected. The diagnosis is made based on clinical presentation. First line therapy for trigeminal neuralgia is with carbamazepine. Refractory cases may require surgical management. Dental X-rays (B) are useful in finding small apical abscesses of teeth but will not have utility in trigeminal neuralgia. A CT scan of the mandible (C) is most useful after acute trauma to diagnose mandible fractures. Trigeminal neuralgia does not result from inflammation so there is no role for prednisone (D) or other steroids.

A patient presents to the emergency department with severe burns from a gas explosion. The burn appears waxy dry and red with blistering. It does not blanch with pressure and is only painful to pressure. Which of the following is the most accurate classification of burn in this patient? Deep partial-thickness burn Full-thickness burn Superficial burn Superficial partial-thickness burn

Correct Answer ( A ) Explanation: This patient's presentation is most consistent with a deep partial-thickness burn. Current designations of burn depth are superficial, superficial partial-thickness, deep partial-thickness, and full-thickness. The term fourth degree is still used to describe the most severe burns, burns that extend into the muscle, bone, or joints. Superficial burns involve only the epidermal layer of skin. They do not blister but are painful, dry, red, and blanch with pressure. This process is commonly seen with sunburns. Partial thickness burns involve the epidermis and portions of the dermis. They are characterized as either superficial or deep. Superficial partial-thickness burns characteristically form blisters and are painful, red, and weeping, and blanch with pressure. Deep partial-thickness burns extend into the deeper dermis and damage hair follicles and glandular tissue. They are painful to pressure only, almost always blister, are wet or waxy dry, and have variable mottled colorization from patchy cheesy white to red. They do not blanch with pressure. These burns invariably cause hypertrophic scarring. Full-thickness burns extend through and destroy all layers of the dermis and often injure the underlying subcutaneous tissue. Burn eschar, the dead and denatured dermis, is usually intact. The eschar can compromise the viability of a limb or torso if circumferential. Full thickness burns are usually anesthetic or hypoesthetic. Skin appearance can vary from waxy white to leathery gray to charred and black. The skin is dry and inelastic and does not blanch with pressure. Vesicles and blisters do not develop. A full-thickness burn (B), superficial burn (C) and superficial partial-thickness burn (D) are not consistent with this clinical picture.

A 32-year-old woman presents with fever and lower abdominal pain. She has a history of pelvic inflammatory disease. Her vitals are T 38.4°C, HR 133, and BP 101/60. On examination, the patient is toxic appearing and has marked lower abdominal tenderness to palpation with rebound and guarding. Pelvic examination reveals cervical motion tenderness, scant discharge, and left adnexal tenderness. The patient's urine beta-hCG is negative. A transvaginal ultrasound is performed and reveals a complex cystic, thick-walled, well-defined mass in the left adnexa. Which of the following is the most appropriate next step in management? Administer ceftriaxone and discharge home with a 14-day course of doxycycline Begin intravenous antibiotics and admit for possible drainage Obtain a CT scan to rule out appendicitis Send a serum beta-hCG to rule out ectopic pregnancy

Correct Answer ( B ) Explanation: A tubo-ovarian abscess (TOA) typically results as a complication of pelvic inflammatory disease (PID) and is most commonly seen in sexually active women. Since it is a complication of PID, patients typically present with lower abdominal and pelvic pain, fever, vaginal discharge, and cervical motion or adnexal tenderness. Pelvic exam may reveal a palpable mass in the adnexa. Ultrasound is the test of choice for suspected TOA, and transvaginal is best for visualizing the adnexa. Ultrasound findings typically include identification of a complex cystic, thick-walled, well-defined mass in the adnexa or retrouterine area. The mass is usually multiloculated with air-fluid levels. If ultrasound is equivocal, a CT scan can aid in the diagnosis. Treatment involves administration of intravenous antibiotics. Some abscesses require surgical drainage. Tubo-ovarian abscesses can result in irreversible tubal and ovarian damage and pose a serious threat to fertility. Although ceftriaxone and doxycycline (A) is the treatment for PID, patients with a TOA require a prolonged course of intravenous antibiotics and possible surgical drainage. This requires inpatient management. Appendicitis should be considered in a febrile woman with abdominal pain, particularly if the pain is right-sided. However, in the clinical scenario above, the ultrasound findings are highly suspicious for a TOA; appendicitis (C) is an unlikely diagnosis. Ectopic pregnancy (D) and TOA can present with similar symptoms. In this clinical scenario, the patient's hCG is negative which excludes ectopic pregnancy.

A 58-year old man is brought to the ED for chest pain that started 30 minutes prior to arrival while he was jogging in the park. Initially, the patient's cardiac monitor shows sinus tachycardia with a rate of 120 beats per minute. However, while you are interviewing the patient in the resuscitation bay, he suddenly becomes pale, pulseless, and the above rhythm is seen on the cardiac monitor. Which of the following is the definitive next step to manage this rhythm? Chest compressions Defibrillation Epinephrine Synchronized cardioversion

Correct Answer ( B ) Explanation: The treatment of ventricular fibrillation depends upon whether the onset was witnessed or unwitnessed. If the cardiac arrest is witnessed (as in this case) and of short duration with an initial rhythm of ventricular fibrillation or tachycardia, the patient should receive immediate defibrillation with 200 joules biphasic (or 360 joules monophasic). Newer biphasic defibrillators are preferred. They have a better first-shock success rate than the older monophasic models have while delivering less electrical current and causing less myocardial cell damage. However, if the downtime is unknown, two minutes of CPR (A) is recommended prior to defibrillation for a shockable rhythm to help "prime" the heart to receive the shock. The main benefit of epinephrine (C) during CPR is derived from its alpha-receptors activity, which produces a direct increase in peripheral vascular resistance. In turn, this leads to a subsequent rise in central aortic blood pressure and an improvement in coronary perfusion pressure during chest compressions. For patients with a shockable rhythm, defibrillation is priority, and epinephrine should be administered only after this and five cycles of CPR have been delivered. After the initial dose, epinephrine can be readministered every three to five minutes. Synchronized cardioversion (D) is reserved for those patients with a palpable pulse.

A 22-year-old man presents to the ED with unilateral throbbing headache. He denies family history of headache, however, he has had this type of headache before. Beginning around age 20 years, he started having 4-6 "flares" a day for 6 weeks at a time. Each flare would last 30 minutes and they mainly occurred at night. Eventually they resolved, but reappeared after his 21st birthday and lasted for about 2 months. He had been asymptomatic until 2 days ago when the headaches returned. Physical examination shows conjunctival injection and rhinorrhea. Which of the following treatments should most likely be initiated first? Acetaminophen, aspirin, and caffeine Oxygen Topiramate Verapamil

Correct Answer ( B ) Explanation: Cluster headaches occur most often in middle-aged men. It is described as a recurrent, unilateral, excruciating periorbital headache that lasts from 15 minutes to 3 hours. Headaches are nonpulsatile and constant, frequently occurring at night. Cluster headaches are associated with ipsilateral conjunctival injection, lacrimation, and nasal congestion. Cluster headaches classically take place in groups over days to weeks, occurring at the same time of day and in the same location. Acute attacks can be treated with 100% oxygen. Other treatments include triptans, ergotamines, intranasal lidocaine, and butorphanol. Topiramate (C) and verapamil (D) are used in the prevention, not abortion, of cluster headache. Acetaminophen-aspirin-caffeine (Excedrin®) (A) is used more commonly in treating migraine, not cluster, headache.

What cardiac complication is associated with hyperthyroidism? Aortic Dissection High-output cardiac failure Pericarditis Ventricular dysrhythmias

Correct Answer ( B ) Explanation: High-output cardiac failure is associated with hyperthyroidism. Hyperthyroidism is caused by an increased circulating level of thyroid hormone. Elevated thyroid hormone levels produce a hypermetabolic state and increased beta-adrenergic activity. Common cardiac complaints include palpitations, dyspnea on exertion and decreased exercise tolerance. New-onset atrial fibrillation is common especially in the elderly. Through beta-adrenergic stimulation, elevated thyroid hormone levels can produce high-output cardiac failure. Dilated cardiomyopathy can result from prolonged tachycardia (or tachydysrhythmias) along with the high output state. Aortic dissection (A) and pericarditis (C) are not associated with hyperthyroidism. Atrial dysrhythmias, not ventricular (D) are associated with hyperthyroidism.

A six-year-old girl presents with dark urine and pedal edema and is found to have an elevated blood pressure. She was treated for strep pharyngitis two weeks ago. Which of the following is the next best step? Obtain a blood count Obtain a urinalysis Obtain antistreptolysin titers Obtain serum complement levels

Correct Answer ( B ) Explanation: The patient in this vignette likely has poststreptococcal glomerulonephritis (PSGN). In any patient with concern for renal disease, a urinalysis should be obtained. In patients with renal disease, urine dipstick results show large amounts of blood and protein. Microscopic analysis usually yields leukocytes and white blood cell casts. Red blood cells may appear dysmorphic and can be present with red blood cell casts. Red blood cell casts may or may not be related to PSGN but is pathognomonic for glomerular disease in general. A complete blood count (A) is not usually helpful in the initial workup for renal disease. In this case, the likely diagnosis is PSGN; however, a urine analysis is necessary in the initial workup for any renal disease. Antistreptolysin titers (C) will aide in the diagnosis once renal disease is established. Complement studies (D) are very useful in the diagnosis of PSGN as the consumption of complement is a component in the pathogenesis of the disease. However, urine studies should be obtained prior to any further workup when renal disease is suspected.

The scaphoid articulates with which of the following bones? Hamate Radius Triquetrum Ulna

Correct Answer ( B ) Explanation: The scaphoid bone is one of the eight carpal bones, which all together with the distal radius and ulna, make up the bones of the wrist. The carpal bones are arranged in two rows of four bones. The proximal row consists of the scaphoid, lunate, triquetrum, and pisiform, while the distal row consists of the trapezium, trapezoid, capitate, and hamate. The wrist has many complex articulations which allow for complex movements, including the radiocarpal joint, the midcarpal joint, and the distal radioulnar joint, which allow wrist flexion, extension, radial deviation (abduction), ulnar deviation (adduction), circumduction, pronation, and supination. The scaphoid and lunate bones articulate proximally with the radius to form the radiocarpal joint. The radius also articulates with the ulna at the distal radioulnar joint. The ulna does not directly articulate with the carpal bones, but rather has an interface with the triangular fibrocartilage complex, known as the articular disk, which binds together the distal ends of the radius, ulna, lunate, and triquetrum. Scaphoid fractures can be radiographically difficult to detect. In 15% of cases, radiographs taken immediately after injury fail to show a fracture line. Therefore, if a scaphoid fracture is suspected, the wrist should be immobilized in a thumb spica splint and follow-up for repeat clinical evaluation and radiographic imaging in 7-10 days. The scaphoid bone does not articulate with the hamate (A), triquetrum (C), or ulna (D).

You are examining an afebrile 78-year-old woman in the emergency department. During cardiac examination, you auscultate a low intensity, low pitch extra heart sound which occurs in early diastole. You do not appreciate any murmurs. Her ECG appears normal. Which of the following is the most likely diagnosis? Bacterial endocarditis Dilated cardiomyopathy Right bundle branch block Tricuspid stenosis

Correct Answer ( B ) Explanation: The third heart sound (S3) is a low-frequency, brief vibration occurring in early diastole, at the end of the rapid diastolic filling period of the right or left ventricle. It is best appreciated at the apex in the left lateral position. It may be a normal variant in patients younger than age 40. After age 40, the presence of an S3 is usually abnormal, and correlates well with ventricular dysfunction, namely volume overload. However, any cause of ventricular dysfunction may be causative: dilated or ischemic cardiomyopathy, conduction abnormalities, left-to-right intracardiac shunts, ischemic heart disease, myocarditis or valvular regurgitation. High output states, such as anemia, thyrotoxicosis or pregnancy, also are causative. Although conduction abnormalities can cause ventricular dysfunction, bundle branch block (C) is unlikely in a patient with a normal ECG. Patients with bacterial endocarditis (A) are typically febrile. These patients usually have tricuspid or mitral valve vegitations. Tricuspid stenosis (D) is a rare complication of rheumatic fever. It is associated with a diastolic murmur.

A 19-year-old woman presents with lower abdominal pain for 5 days. You consider pelvic inflammatory disease as a diagnosis. Which of the following is a likely contributor for this condition? Age over 25 years Barrier contraception Multiple sexual partners Pregnancy

Correct Answer ( C ) Explanation: Although there are no definitive risk factors for pelvic inflammatory disease (PID), there are several contributors. Multiple sexual partners increases the chances for developing PID. Other risk factors include earlier age at first intercourse; instrumentation, including induced abortion and intrauterine device insertion; and the period immediately following menses. Most cases of PID occur in women <25 years of age (A). Females 15-24 have increased numbers of sexual partners, have a cervical barrier more easily breached by pathogens, often have less frequent use of barrier contraception (B) and tend to seek health care later. Pregnancy (D) confers protection from PID after the 1st trimester when the uterine cavity is obliterated by the pregnancy. However, PID can occur in the 1st trimester.

A woman with chronic constipation and poorly controlled diabetes mellitus presents with acute anal pain that is exacerbated by defecation. Inspection of the anal border reveals no abnormalities. Which of the following is the most likely diagnosis? Anal fistula Perianal hematoma Perirectal abscess Thrombosed external hemorrhoid

Correct Answer ( C ) Explanation: Any abscess, arising from an infection of the anal sinuses, is called an anorectal abscess, or perianal or perirectal abscess. Sporadic onset is common. There are four main types of anorectal abscesses: perianal, ischiorectal, intersphincteric and supralevator. Dull or throbbing anal pain, usually acute in onset, is the main symptom. It usually worsens prior to, and lessens after, a bowel movement. Direct visual inspection may miss a deep abscess, therefore, it is important to palpate for such a possibility. Imaging is typically not necessary but CT can be obtained if there is concern for a deep abscess. Treatment mainstays are antibiotics and surgical drainage. Anal fistula (A) is an abnormal connection between the epithelium of the anal canal and the perianal skin. Examination usually reveals the surface opening typical of a fistula, occurring with or without drainage. A perianal hematoma (B) occurs on the anal border. It typically can be seen during examination as a blue-tinged bulge, which is soft, smaller and mobile. If left untreated, it can progress to a thrombosed external hemorrhoid (D), which is hard and larger and is commonly visualized on physical exam.

Which of the following is true regarding attention deficit hyperactivity disorder? Characterized by speech delay, poor eye contact, and lack of stranger anxiety More prevalent in girls than boys Most frequently diagnosed behavioral disorder in children Symptoms most commonly begin in adolescence

Correct Answer ( C ) Explanation: Attention-deficit/hyperactivity disorder (ADHD) is the most frequently diagnosed behavioral disorder of childhood, with a prevalence of 4% to 12%. At least 10% of behavior problems seen in a general pediatric practice are caused by ADHD. ADHD is characterized by a triad of symptoms including inattention, impulsivity, and hyperactivity. Symptoms must be present in 2 areas of social interaction (home and school); must have been present prior to age 12 years; must have persisted for longer than 6 months; and must be maladaptive or inappropriate for the child's developmental stage. Research suggests that ADHD has a central nervous system (CNS) basis; however, no specific etiology has been discovered. Various brain imaging studies of ADHD patients have demonstrated abnormalities of brain metabolism, supporting the validity of ADHD as a disorder. However, the strongest evidence of validity has been course prediction and treatment response to medication. There is no independent valid test to determine that a child has ADHD. The diagnosis can only be obtained reliably by using well-established diagnostic assessment methods. This involves using the standardized diagnostic criteria of the American Psychiatric Association's Diagnostic and Statistical Manual of Mental Disorders (or DSM). Speech delay, poor eye contact, and lack of stranger anxiety are characteristic of autism (A), not ADHD. ADHD is more prevalent in boys (B) than girls. Symptoms must have been present (D) before the age of 12.

In which of the following ways does an essential tremor differ from the tremor of Parkinson's disease? Essential tremor can be treated with dopamine agonists Essential tremor is a pill rolling tremor Essential tremor is exacerbated by action Essential tremor is unilateral

Correct Answer ( C ) Explanation: Essential tremor is usually symmetric and exacerbated by action, whereas the tremor of Parkinson's disease is usually asymmetric and at rest. Essential tremor affects up to 5% of the general population after the age of 60. Essential tremor is often inherited in an autosomal dominant fashion. The age of onset may be as early as the first or second decade of life, but senile tremor is frequently delayed until the mid-60s. Patients first become aware of mild postural and action tremor in the hands, which is indistinguishable from an enhanced physiologic tremor, and may result in minimal functional impairment for many years until it gradually progresses. Treatment of essential tremor does not influence the course of the illness, and therefore is justified only when the tremor interferes with function. At least 50% of patients note improvement or complete amelioration of tremor following the ingestion of a small amount of ethanol. First-line drug treatment includes trials of a non-selective beta-adrenergic blocker (e.g., propranolol) or primidone. Parkinson's disease (D) consists of a tremor that is unilateral, described as "pill rolling" (B), and can be treated with a dopamine agonist (A).

A 31-year-old nurse presents to the emergency department after being stuck with a blood-filled hollow bore needle during a resuscitation. She discovers the patient has hepatitis B. That same day she has her blood drawn. Which of the following serologic markers indicates prior hepatitis B immunization? Hepatitis B core antibody (anti-HBc) Hepatitis B envelope antigen (HBeAg) Hepatitis B surface antibody (anti-HBs) Hepatitis B surface antigen (HBsAg)

Correct Answer ( C ) Explanation: Presence of the hepatitis B surface antibody (anti-HBs) immediately postexposure indicates prior immunization. Although anti-HBs may also be seen with acute infection, its appearance is associated with a postexposure lag time of roughly two months. A person negative for HBsAg but positive for anti-HBs either has cleared an infection or has been vaccinated previously. The hepatitis B core antibody (anti-HBc) (A) is the earliest antibody to develop in response to acute hepatitis B virus (HBV) infection, appearing predominantly as IgM anti-HBc at about six to eight weeks after infection. The anti-HBc typically persists for life, but after about six months the total anti-HBc mainly consists of IgG rather than IgM. Shortly after the appearance of the HBsAg in an acute infection, the hepatitis B e antigen (HBeAg) (B) appears. In general, the presence of HBeAg is associated with rates of viral replication and increased infectivity; however, some variants of hepatitis B virus do not produce the e antigen. During the natural course of an infection, the HBeAg may be cleared, and antibodies to the e antigen (anti-HBe) will arise immediately afterward. This conversion is usually associated with a dramatic decline in viral replication and infectivity. The hepatitis B surface antigen (HBsAg) (D) is used to screen for the presence of acute hepatitis B infection. It is the first detectable viral antigen to appear during infection. However, early in an infection, this antigen may not be present; it may also be undetectable later if the infection has been adequately cleared by the host. During this window in which the host remains infected but is successfully clearing the virus, IgM antibodies to the hepatitis B core antigen (anti-HBc IgM) may be the only serological evidence of disease.

Which of the following should be assessed in the physical exam of a patient with essential hypertension? Costovertebral angle tenderness Deep tendon reflexes Ocular fundus Pupillary response

Correct Answer ( C ) Explanation: Hypertension is defined as a systolic blood pressure of 140 to 159 mm Hg or a diastolic blood pressure of 90 to 99 mm Hg on two or more occasions. Hypertension is commonly seen in the primary care setting and risk factors include family history of hypertension, obesity, advancing age, African-American race, physical inactivity, high sodium diet, and diabetes. When taking the history of a patient with hypertension, questions should include duration of the diagnosis, previous treatment, and aggravating factors such as alcohol consumption, smoking, use of nonsteroidal anti-inflammatory agents and other prescription medications. The purpose of the physical exam is to assess for end-organ damage and cardiovascular disease as well as potential causes of secondary hypertension. Physical exam should include an accurate reading of blood pressure along with an assessment of general appearance, heart, lungs, neck, abdomen, and extremities. A fundoscopic exam of the eyes should be used to evaluate for hypertensive retinopathy including hemorrhage, papilledema and cotton wool spots. Costal vertebral angle tenderness (A) is assessed to rule out pyelonephritis or renal stones. Deep tendon reflexes (B) are evaluated in patients as part of a complete neurological exam. Neurological abnormalities seen in patients with hypertension include visual disturbance, focal weakness or confusion. Pupillary response (D) is assessed as part of cranial nerve testing in patients with neurological defects.

What concomitant electrolyte disorder should be suspected in a patient with hypokalemia who fails to respond to potassium repletion? Hypercalcemia Hypernatremia Hypomagnesemia Hyponatremia

Correct Answer ( C ) Explanation: Hypokalemia is associated with hypomagnesemia and successful potassium repletion requires magnesium replacement. Hypokalemia is the most frequently encountered electrolyte abnormality in the Emergency Department. Typically, it is defined as a serum potassium <3.5 mEq/L. It is usually asymptomatic but can present with nonspecific symptoms like weakness, fatigue, muscle cramping and paresthesias. At very low levels, cardiac dysrhythmias, rhabdomyolysis and hypotension can occur. Although there are a number of causes of hypokalemia, iatrogenic from medication use is one of the most common. Effective potassium repletion requires magnesium to act as a cofactor in transport of potassium across cell membranes. Without magnesium, administered potassium will stay in serum and be excreted in urine. Patients should receive 0.5 grams/hour of magnesium sulfate with potassium repletion. Although patients often experience multiple electrolyte abnormalities, hypernatremia (B), hypercalcemia (A) and hyponatremia (D) are not associated with failure to replete potassium levels.

A previously healthy 3-week-old male born full-term presents with sudden onset bilious emesis. There appears to be abdominal tenderness and distention on exam. Which of the following is the most accurate statement regarding the suspected diagnosis? Imaging will often show air in the biliary tree It is associated with maternal use of erythromycin during late pregnancy and breastfeeding The diagnosis is confirmed on upper GI series with failure of the duodenal-jejunal junction to cross midline The primary metabolic abnormality is hypochloremic metabolic alkalosis

Correct Answer ( C ) Explanation: Intestinal malrotation refers to a spectrum of congenital anomalies which involve failure to develop normal rotation of the intestinal tract. The formation of normal bowel in the embryo involves a rotation of elongated intestine into the abdominal cavity. The proximal small intestine develops a C-shaped contour, with the duodenum fixing to the left of the midline at the ligament of Treitz. The cecum undergoes counter-clockwise rotation to end up in the right lower abdomen. Incomplete rotation results in inadequate fixation of the intestinal mesentery. Bands may be formed which cause incomplete intestinal obstruction. If there is inadequate fixation of the small bowel, the intestine can twist on the axis of the superior mesenteric artery. The majority of patients present within the first year of life and of those, most present within the first month of life with symptoms of obstruction. The classic presentation is an otherwise healthy infant who develops sudden onset bilious vomiting. Patients may also present with hematochezia which indicates bowel ischemia and has a poorer prognosis. Intestinal malrotation is usually diagnosed with an upper GI series and confirmed by observing failure of the duodenal-jejunal junction to cross midline. Contrast studies often show a spiral or corkscrew appearance of the distal duodenum in cases of volvulus. Management of uncomplicated malrotation with or without volvulus is typically laparoscopic surgery (the Ladd's procedure), although patients with significant bowel ischemia may require an open approach. Recurrent volvulus after surgical repair is rare, but possible. After the first year of life, patients with malrotation present with more varied symptoms such as intermittent abdominal pain, diarrhea, and bilious or non-bilious emesis, and bilious vomiting is not commonly the predominant symptom. As a result patients are often misdiagnosed as having simple gastroenteritis, cyclic vomiting, or psychological disorders. Because prolonged volvulus may lead to more serious consequences such as malabsorption, failure to thrive, and intestinal ischemia, a high index of suspicion is required. Air in the biliary tree (A) is commonly seen in necrotizing enterocolitis (NEC). NEC most commonly occurs in infants born pre-term. Maternal use of erythromycin (B) during pregnancy or while breastfeeding is associated with development of pyloric stenosis, but it has not been linked to malrotation. Hypochloremic metabolic alkalosis (D) is associated with pyloric stenosis, not intestinal malrotation.

Which of the following antibiotics may precipitate hemolysis in a patient with G6PD deficiency? Amoxicillin/Clavulanate Cephalexin Nitrofurantoin Streptomycin

Correct Answer ( C ) Explanation: Once the diagnosis of G6PD deficiency is established, patients must be counseled to avoid drugs, chemicals, and foods known to precipitate hemolysis. Although there are some medications classically connected to hemolysis in G6PD patients, there are a number of medications for which there is considerable confusion as to whether they are safe in G6PD-deficient patients. In an attempt to better clarify which are truly high-risk drugs, investigators in Israel performed a comprehensive literature search and categorized drugs according to how much evidence there is in the literature to contraindicate their use. They found only seven medications for which there is solid evidence to prohibit their use in G6PD-deficient patients: dapsone, methylthionine chloride (methylene blue), nitrofurantoin, phenazopyridine, primaquine, rasburicase, and tolonium chloride (toluidine blue). Their review found no substantial evidence to absolutely contravene the use of other medications in normal therapeutic doses. G6PD deficiency is the most prevalent human enzyme deficiency in the world, affecting an estimated 350 to 400 million people. The geographic distribution of G6PD deficiency coincides with the geographic distribution of endemic malaria, implicating a survival benefit. The highest prevalence of G6PD deficiency is in sub-Saharan Africa, followed by the Middle East, Mediterranean Europe, and Southeast Asia. Most people with G6PD deficiency have no clinical symptoms and are not anemic. In fact, the majority of affected individuals live out their lives unaware of their status. Diagnosis typically occurs when an episode of acute hemolysis is triggered by exposure to oxidant drugs, infection, or ingestion of fava beans. Unusual presentations include hemolysis precipitated by complications of diabetes, myocardial infarction, and strenuous physical exercise. There is no contraindication to the use of amoxicillin/clavulanate (A), cephalexin (B), streptomycin (D), in a patient with G6PD.

Which of the following is the most common cause of post-partum hemorrhage? Laceration to the cervix Retained placenta Uterine atony Uterine rupture

Correct Answer ( C ) Explanation: Postpartum hemorrhage is defined as a loss of > 500 mL of blood after delivery. It is divided into early (< 24 hours) and late (> 24 hours) hemorrhage. Early postpartum hemorrhage can have brisk bleeding and shock can develop rapidly. Uterine atony is the most common cause of postpartum hemorrhage. Uterine atony occurs for a multitude of reasons including the over distended uterus (multiple gestation), augmented labor (oxytocin), high parity, and quickening placental delivery. The uterus should be palpated immediately after delivery. An enlarged and "boggy" uterus is seen with uterine atony. If this is palpated, then vigorous uterine massage should be performed to help prevent atony. A slow administration of oxytocin can be administered with uterine massage to help prevent atony, as well. Ergot derivatives and prostaglandins can also be given if the bleeding is unresponsive to uterine massage and oxytocin. Fluid resuscitation should begin if there is severe blood loss. Surgery may be required if the bleeding is unresponsive to the treatments above. Cervical laceration (A) occurs in greater than half of all vaginal deliveries. Most of these are under 0.5 cm. Small lacerations, up to 2 cm, heal on their own without complication. A larger or deeper cervical laceration should be suspected if there is bleeding with a firm uterus. The cervix should be visualized completely after delivery to check for lacerations and if bleeding profusely should be surgically repaired. A retained placenta (B) is a rare cause of post-partum hemorrhage. The placenta should be thoroughly examined after delivery and if a piece is missing, the uterus should be explored and portions of the placenta that are retained should be removed. Uterine rupture (D) has a multitude of causes including previous uterine scarring from C-section or other uterine surgery, including surgeries involving the cervix. This is the most common cause of uterine rupture. The uterus can also rupture if overstimulated which can occur with the use of oxytocin induction or augmentation. The treatment is surgical and hysterectomy may be required if the rupture is life-threatening.

Which of the following is true regarding sudden unexpected death of infancy (SUDI)? Most cases occur in infants younger than two months old Normal-term infants have a higher incidence of SIDS than preterm infants Peak incidence is between two and four months of age The diagnosis is usually made in the emergency department

Correct Answer ( C ) Explanation: Some 95% of sudden unexpected death of infancy (SUDI) infants die before six to eight months, with a peak occurring between two and four months of age. SUDI may occur at any time during the first two years of life, but it is rare in children younger than one month of age and in those older than one year of age. African American, Native American, and Alaskan Native infants have rates 2- to 3-fold greater than the national average. Male babies and multiple births are also risk factors for SUDI. Most cases occur (A) in infants younger than six to eight months, with a peak occurring between two and four months. Approximately 20% of all SUDI cases occur in the preterm (B) population. The diagnosis of SUDI (D) is made at autopsy after postmortem evaluation fails to reveal another cause of death. Autopsies of SUDI victims demonstrate the effects of chronic hypoxemia, but no specific findings are pathognomonic of SUDI.

What are the components of the "female athlete triad"? Anemia, iron deficiency, menorrhagia Delayed puberty, ligamentous injuries, weight loss Low bone density, menstrual dysfunction, low energy availability menstrual dysfunction, hirsutism, ovarian cysts

Correct Answer ( C ) Explanation: The female athlete triad is defined by the presence of low bone density (can be normal), menstrual dysfunction (ranges from normal ovulatory cycles to luteal phase defects and anovulatory eumenorrhea to amenorrhea), and low energy availability (ranges in athletes from inadvertent undereating to disordered eating to a frank eating disorder). It is important to recognize the precursors to the development of the female athlete triad when they may be more amenable to treatment, resulting in less severe long-term sequelae. Caloric deficiency is the primary cause of amenorrhea in athletic women, and treatment should focus on the restoration of normal calorie intake. Athletic amenorrhea is caused by hypothalamic-pituitary axis suppression and is a diagnosis of exclusion. Bone mineral density is adversely affected by menstrual dysfunction and, although treatment with hormone replacement (e.g., oral contraceptives) should be considered, this does not fully address the mechanisms of bone loss. It is never normal or desirable for a female athlete to cease menstrual function, and this should not be seen as a marker of adequate training. Exercise alone should not be blamed for menstrual dysfunction. Many women engage in exercise to control body weight and improve exercise capacity. Amenorrhea only occurs when there is a relative caloric deficiency due to inadequate nutritional intake relative to the amount of energy expended. Although anemia, iron deficiency and menorrhagia (C) can all be seen in any woman who has heavy menstrual cycles, it is not called the female athlete triad. Delayed puberty, ligamentous injuries and weight loss (D) can all be seen individually in the exercising woman, but these components are not called the female triad. Many of these women suffer from stress fractures as opposed to ligamentous injuries secondary to osteoporosis. Amenorrhea, hirsutism and ovarian cysts (B) are all components of polycystic ovarian syndrome.

A mother brings her 22-month-old child to the ED after noticing bloody bowel movements. The child has a normal birth history. Mom denies any pain associated with these episodes, but she states that the child has become increasingly pale with decreased energy. His vital signs include a blood pressure of 95/60 mm Hg, heart rate of 140 beats per minute, respiratory rate of 24 breaths per minute, and oxygen saturation of 98% on room air. Abdominal examination is unremarkable, but his stool is guaiac positive. Which of the following is most likely to confirm the diagnosis? Abdominal ultrasound Air contrast enema Technetium-99m pertechnetate scintiscan Upper GI series via nasogastric tube

Correct Answer ( C ) Explanation: The patient's presentation is classic for Meckel's diverticulum, the most common congenital malformation of the GI tract, and follows the rule of 2s: the diverticulum is 2 cm wide and 2 inches long and usually located within 2 feet of the ileocecal valve. The condition occurs in 2% of the population, and only 2% of affected patients ever become symptomatic. Many contain heterotropic tissue, the most common of which are gastric mucosa and pancreatic acini. The classic presentation is painless rectal bleeding in a boy younger than five years. Diagnosis is made by radionuclide scanning (technetium-99m pertechnetate scintiscan), also called a Meckel's scan. The test is highly specific. An abdominal ultrasound (A) is used to diagnose pyloric stenosis which presents as nonbilious projectile vomiting within the first weeks of life. An air contrast enema (B) can be both diagnostic and therapeutic in the evaluation of intussusception which classically presents with intermittent abdominal pain and vomiting. An upper GI series (D) may show the classic corkscrew sign (dilation of the duodenum) in patients with intestinal malrotation and midgut volvulus, a disorder that typically presents with bilious vomiting and abdominal pain.

Which of the following statements is true regarding this condition? Associated with recent infection Most commonly occurs in boys ages four to eight years Pain can be referred to the knee Radiographs can be normal

Correct Answer ( C ) Explanation: The radiograph represents slipped capital femoral epiphysis (SCFE), a disorder seen in early childhood characterized by chronic slipping of the femoral epiphysis of the hip. SCFE is the most common cause of hip disability in adolescents. Clinically, the child develops hip pain referred to the thigh or knee. Internal rotation of the hip induces pain. It is more common in obese, African American males. Other risk factors include hypothyroidism, growth hormone deficiency, and renal osteodystrophy. Transient tenosynovitis (A) is a cause of pediatric limp typically preceded by a viral infection. Legg-Calve-Perthes disease (B) is an idiopathic avascular necrosis that most commonly occurs in boys aged four to eight years, whereas SCFE most commonly occurs in obese boys with an average age of 12 to 16 years. Radiographs are not normal (D) and will demonstrate epiphyseal slippage. In subtle cases, MRI may be needed for confirmation.

A 44-year-old businessman is having worsening hearing loss in his right ear over several months. He says it sounds as though everyone is "mumbling." He also endorses feeling persistently off-balance. He denies tinnitus. Which of the following clinical interventions is most appropriate for this patient? Education on using the Epley maneuver Prescribe acetazolamide and recommend a low sodium diet Prescribe oral prednisone tapered over 10 days Schedule an auditory canal MRI

Correct Answer ( D ) Explanation: An auditory canal MRI is the next best step in managing this patient, who is exhibiting signs and symptoms of an acoustic neuroma. Acoustic neuromas are schwannomas of the eighth cranial nerve that arise within the internal auditory canal. Though benign, they gradually grow and can eventually impair hearing as well as compress the pons, causing hydrocephalus. Patients are typically diagnosed with acoustic neuromas once hearing deficits occur. Since these lesions usually occur unilaterally, the typical hearing loss pattern is unilateral with a steady deterioration in speech discrimination. While sudden vertigo is unlikely, a persistent disequilibrium is common. Tinnitus occasionally occurs. A contrast-enhanced MRI of the auditory canal will allow for visualization of the lesion. The decision to pursue treatment must be based on the individual patient's tumor size, age, and overall health. Management options include microsurgical excision, stereotactic radiotherapy, or simply observation. Education on using the Epley maneuver (A) is the appropriate intervention to manage benign paroxysmal positioning vertigo (BPPV). However, the classic presentation is BPPV is sudden onset vertigo, usually related to swift head movements. Unilateral hearing loss is not caused by BPPV. Prescribing acetazolamide and recommending a low sodium diet (B) is the correct treatment for hearing loss and vertigo caused by Ménière's Syndrome, which usually causes episodes of vertigo lasting from 20 minutes to several hours, as well as low-frequency hearing loss and roaring tinnitus. The duration of this patient's symptoms do not fit those of Ménière's Syndrome. Promptly prescribing an oral prednisone taper (C) would be necessary in managing any sudden sensory hearing loss to prevent patients from developing lasting deafness. This patient, however, had been having worsening hearing loss over several months, and corticosteroids appear to not be effective for sudden sensory hearing loss after 6 weeks of impairment has passed.

A woman presents with dyspnea on exertion. Cardiac examination reveals an apical late diastolic murmur. You also notice pitting edema in both her legs. She undergoes echocardiographic testing. Which of the following abnormalities would you most expect to see on the echocardiogram? Decreased trans-mitral mean pressure gradient Increased opening of the mitral valve leaflets Left atrial hypertrophy Right ventricular hypertrophy

Correct Answer ( D ) Explanation: One of the diastolic murmurs is mitral stenosis, which is associated with rheumatic heart disease and endocarditis. This murmur is best appreciated at the apex with the bell, especially when the patient is placed in the left lateral decubitus position. It is characterized as a late diastolic, low frequency rumble which does not radiate. It is commonly associated with a left sternal border thrill. The intracardiac narrowing of this condition hampers outward flow from the left atrium, causing a "pressure back-up" into the pulmonary circulation and ultimately the right heart. Subsequently, right ventricular hypertrophy, and possibly right heart failure, are frequent associated findings. Symptoms of mitral stenosis include fatigue, weakness, dyspnea on exertion, orthopnea, palpitations and chest pain. If the condition is chronic, and right heart failure is present, peripheral edema, ascites and hepatomegaly may be present. Pregnancy and exercise can exacerbate the fatigue and weakness encountered in someone with mitral valve stenosis. An increased, not decreased, trans-mitral mean pressure gradient (A) is seen during Doppler echocardiography of mitral stenosis. It is the gold standard in diagnosing the severity of this valve disease. Decreased, not increased, mitral valve opening (B) is seen with mitral stenosis, whereas, increased mitral valve motion may be present in mitral valve prolapse. Left atrial dilation, not hypertrophy (C), is commonly found with mitral stenosis.

A 58-year-old African-American man with a history of hypertension presents to your office for his annual exam. His BP is 145/95 and HR is 75. He is taking lisinopril as previously prescribed. Based on the most recent hypertension guideline, which of the following is the most appropriate next step in his management? Continue the lisinopril and start the patient on an angiotensin receptor blocker Continue the lisinopril and start the patient on hydrochlorothiazide Discontinue the lisinopril and start the patient on an angiotensin receptor blocker Discontinue the lisinopril and start the patient on hydrochlorothiazide

Correct Answer ( D ) Explanation: The Report From the Panel Members Appointed to the Eighth Joint National Committee (JNC 8) provides the most updated guideline for treatment of hypertension. One change from the previous guideline, JNC 7, is the differentiation of drug therapy recommendations based on race. For the general African-American population, initial therapy includes a thiazide-type diuretic or calcium channel blocker. For the general non-black population, initial therapy includes a thiazide-type diuretic, calcium channel blocker, angiotensin receptor blocker, or ACE-inhibitor. ACE-inhibitors (A and B) are not recommended as first line treatment for the general African-American. Angiotensin receptor blockers (C) are also not first line treatment for this patient because of his race.

A 59-year-old man has been experiencing left sided hearing loss over the past several months and he is no longer able to listen to a telephone on his left side. He also has left side tinnitus and feels off-balanced when walking. An audiogram shows high tone hearing loss in the left ear and a 30% word discrimination score. Which of the following is the most likely diagnosis? Labyrinthitis Meniere's disease Presbycusis Vestibular schwannoma

Correct Answer ( D ) Explanation: Vestibular Schwannoma usually presents with a triad of progressive ipsilateral hearing loss, ataxia, and tinnitus. It is a tumor of the vestibular portion of the acoustic (VIII) cranial nerve and has historically been referred to as an acoustic neuroma (although this term is a misnomer). A typical finding is ipsilateral sensorineural hearing loss in the high tones and a poor word discrimination score (hence the frequent complaint of not being able to use a telephone in that ear). Diagnosis is made with neuroimaging (contrast enhanced MRI is most common) and treatment options include: 1) continued observation with periodic imaging 2) radiosurgery 3) surgical resection. Meniere's disease (B) is a disorder of endolymphatic hydrops. It presents with a similar triad of hearing loss, tinnitus, and vertigo. Some key distinctions include: 1) recurrent bouts of severe and often disabling vertigo (as opposed to ataxia) and 2) hearing loss will come and go and is typically in the low tones (as opposed to the high tones). Labyrinthitis (A) is inflammation of the inner ear usually caused by a viral infection and often follows an upper respiratory infection. The vertigo and hearing loss have an acute onset and are self-limited to a few weeks. Treatment is symptomatic in most cases. Presbycusis (C) is age-related hearing loss that typically begins past the age of 65 and is almost always bilateral affecting both ears equally.

What is the characteristic sign of duodenal obstruction on abdominal radiograph?

Double-bubble sign. Malrotation occurs in infants with sudden onset of bilious vomiting, examination shows hemodynamic instability and distension. Need upper G.I. series, which shows that duodenal-jejunal junction does not cross midline. Surgery is Ladd procedure.

Boy, 2, unvaccinated. Was in his usual state of health playing alone in his room when mom heard a coughing fit. Since then, choking and coughing, with inspiratory stridor. Significant work of breathing. Cause? Acute asthma. Croup from parainfluenza. Epiglottitis from H. influenzae. Foreign body aspiration.

Foreign body aspiration. The child's sudden onset of respiratory distress in an unobserved setting is suggestive of a foreign body aspiration. The child's inspiratory stridor is indicative of an upper airway obstruction, and he most likely has a laryngotracheal foreign body aspiration. Aspiration into the bronchi, and especially the right mainstem bronchus, is significantly more common than laryngotracheal aspiration. However, objects with sharp or irregular edges are more likely to become lodged in the upper airway. Children with laryngotracheal foreign body aspiration require immediate medical attention for airway stabilization and removal of the foreign body. Notably, if a child is able to cough, the airway obstruction is partial. A blind finger sweep of the mouth is not recommended, as this may lead to dislodgment of the foreign body and subsequent complete airway obstruction. Radiographic evaluation of the neck is recommended, but only approximately ten percent of foreign bodies are radio-opaque. Instead the radiograph may show subglottic swelling. However, radiographs are insufficient to rule out a foreign body. If the diagnosis is suspected, should be further evaluated by bronchoscopy. Acute asthma (A) can present with respiratory distress but is a more progressive process than what is seen in a foreign body aspiration. Acute asthma is not associated with inspiratory stridor. Croup secondary to parainfluenza virus (B) is a common cause of upper airway obstruction in children. In contrast to laryngotracheal foreign body aspiration, croup has a more insidious onset. It typically begins with symptoms of upper respiratory tract infection prior to progressing to a barking cough, hoarseness, or stridor. Epiglottitis secondary to H.influenzae (C) is always a consideration in children with acute, severe upper airway obstruction. It is especially important to maintain a high index of suspicion in unvaccinated children. Many of the symptoms of epiglottitis are shared with laryngotracheal foreign body aspiration. However, children with epiglottitis are more likely to have fever and appear toxic.

Do girls or boys develop SCFE at a younger age?

Girls typically present at a younger age (10 to 14 years) as compared to boys (12 to 16 years). Slipped capital femoral epiphysis occurs on the left more often. May see abnormal Klein line and Bloomberg's sign. Should make non-weightbearing and consult orthopedics.

If a previously vaccinated hospital worker tests negative for hepatitis B surface antibody and is exposed to hepatitis B, what is the recommended post-exposure prophylaxis?

Hepatitis B immune globulin and hepatitis B vaccine. Surface antibody is the only sign of vaccination.

Which of the following is the first line treatment for acute pericarditis? Citalopram. Ibuprofen. Ketorolac. Prednisone.

Ibuprofen. If there is a specific etiology of the pericarditis, then therapy should be directed at that etiology. Otherwise, nonsteroidal anti-inflammatory drugs (NSAIDs) are the mainstay of therapy for acute pericarditis. Ibuprofen has the best side effect profile, but other NSAIDs should be equally effective.The majority of patients have prompt resolution of symptoms without recurrent pericarditis when treated with NSAIDS alone. However, when used as an adjunct to NSAID therapy, colchicine reduces symptoms, decreases the rate of recurrent pericarditis, and is generally well tolerated. Citalopram (A) is a serotonin-reuptake inhibitor (SSRI) and has no role in the treatment of acute pericarditis. Ketorolac (C) is an NSAID with an excellent analgesic profile, but it has poor anti-inflammatory effects. Therefore, it is not recommended for the treatment of acute pericarditis. Prednisone (D) is a corticosteroid with anti-inflammatory properties that has use limited to the treatment of refractory pericarditis (patients who are intolerant or unresponsive to NSAIDS and/or colchicine). There is a risk of relapse of symptoms when the steroids are tapered or stopped.

Woman, 39. History Hashimoto. Now with hair loss. Perfectly smooth bald patches on scalp with few short hairs growing in each patch. Treatment? Intralesional triamcinolone. Iron supplementation. Prescribe topical minoxidil. Refer for psychiatric counseling.

Intralesional triamcinolone. Intralesional corticosteroids, such as triamcinolone, are commonly effective for managing alopecia areata, which this patient appears to have. Alopecia areata is thought to be an immunologic process by which patients develop patches of smooth, nonscarring hair loss on their scalp that may progress to include facial hair (alopecia totalis) or involve all body hair (alopecia universalis). These patches often contain growth of several 2-3 mm hairs termed "exclamation hairs." Alopecia areata may occur in patients with a history of autoimmune-related disorders, such as Hashimoto thyroiditis, Addison disease, vitiligo, or pernicious anemia. Intralesional corticosteroids usually offer some resolution of the hair loss. Systemic steroids may be used in severe cases, though hair loss generally resumes once they are discontinued. Patients should be reassured that this disease usually resolves spontaneously regardless of treatment, and that 80% of patients with focal disease will have complete hair regrowth. However, the psychological impact of alopecia areata can be significant and patients may benefit from a referral to local supportive groups. Prescribing iron supplementation (B) is not commonly effective for alopecia areata. This intervention may be appropriate in patients with hair loss due to telogen effluvium in whom iron deficiency is thought to be an underlying cause. Prescribing topical Minoxidil (C) will not be the most effective intervention for alopecia areata. Rather, this could be considered in patients with androgenetic pattern baldness which typically causes thinning at the crown and "widow's peak" of the scalp. A referral for psychiatric counseling (D) should be considered if depression or anxiety results because of alopecia areata. However it will not likely improve the hair loss itself. Counseling as sole therapy may be more beneficial in patients with hair loss due to trichotillomania, which causes irregular patches of baldness as patients cope with an uncontrollable compulsion to pulling out their own hair.

You note decreased breath sounds at the left lung base, concerning for small pleural effusion. Which view is most likely to show a small pleural effusion? Lateral. Lateral decubitus left side down. Lateral decubitus right side down. Posterior-anterior (PA).

Lateral decubitus left side down. Classic physical signs of a pleural effusion include diminished breath sounds, dullness to percussion, decreased tactile fremitus, and occasionally a localized pleural friction rub. Chest radiograph confirms the suspicion of pleural effusion. The classic radiographic appearance of a pleural effusion is blunting of the costophrenic angle on the upright chest radiograph. With larger pleural effusions, the hemidiaphragm is obscured, and an upwardly concave meniscus may be seen because the pleural fluid has a tendency to layer higher laterally than centrally. Small pleural effusions are most easily detected on a lateral decubitus film with the affected side down (left side in the above clinical scenario). Accumulations as little as 5 to 50 mL of fluid can be detected with this view. On a PA (D) or AP view, 250 to 500 mL of pleural fluid is required before there is radiographic demonstration. A lesser amount may be visible on the lateral projection (A) in the posterior costophrenic gutter. Pleural effusions are more difficult to detect in lateral decubitus with the affected side up (C).

Girl, 17. Vaginal discharge. Yellow discharge with pruritus. Sexually active for past 6 months, 1 partner. Also has I.U.D. placed 6 months ago. Exam shows white vaginal discharge with strong odor. pH is 5 with clue cells on wet mount. Medical student asks you what causes this (BV). Acquisition of Trichomonas vaginalis. Less Lactobacillus. More Gardnerella vaginalis. More Candida albicans.

Less Lactobacillus (occurs first and is therefore considered the cause). The patient has vaginal discharge due to bacterial vaginosis. It represents a complex change in the vaginal flora brought about by the reduction in concentration of the dominant lactobacilli. These lactobacilli produce hydrogen peroxide that is important in preventing overgrowth of anaerobes that are normally present in the vaginal flora. The loss of lactobacilli results in the increase of vaginal pH and massive overgrowth of anaerobes. These anaerobes produce large amounts of proteolytic carboxylase enzymes that break down vaginal peptides into amines. Common anaerobes include Gardnerella vaginalis, Ureaplasma sp., and Mycoplasma sp. Clinical features include off-white, thin, and homogeneous vaginal discharge with "fishy" odor. Diagnosis is based on the presence of at least three of Amsel criteria: characteristic vaginal discharge, pH ˃ 4.5, positive whiff test, and clue cells on wet mount. Treatment is indicated for relief of symptoms using metronidazole or clindamycin administered either orally or intravaginally. Acquisition of Trichomonas vaginalis (A) is not correct since this organism causes trichomoniasis. Trichomoniasis manifests with yellow-green discharge, strawberry cervix, vaginal pH > 5 and motile trichomonads on wet mount. Increase in concentration of Gardnerella vaginalis (C) can lead to bacterial vaginosis, but this is not the main cause. Gardnerella vaginalis-causing bacterial vaginosis only results after the decrease in lactobacilli. Overgrowth of Candida albicans (D) is also not correct since this causes Candida vulvovaginitis that presents with thick, curdy vaginal discharge, vulvar fissures, and yeast or pseudohyphae on KOH specimen.

Though most acoustic neuromas form unilaterally, which genetic condition should be suspected in patients with bilateral acoustic neuroma formation?

Neurofibromatosis 2.

Girl, 5. Bilateral itching, tearing, and redness of eyes after new family cat 4 days ago. Moderate stringy discharge noted. Normal visual acuity bilaterally. Treatment? Erythromycin ophthalmic ointment. Olopatadine ophthalmic drops. Prednisolone acetate ophthalmic suspension. Sulfacetamide ophthalmic solution.

Olopatadine ophthalmic drops. The patient has allergic conjunctivitis and should be started on olopatadine ophthalmic drops. Olopatadine is a mast cell stabilizer and antihistamine. Allergic conjunctivitis typically presents with bilateral eye itching, tearing, redness, and stringy discharge with occasional photophobia and vision loss. Allergic conjunctivitis can be differentiated from viral conjunctivitis by its predominant pruritus opposed to the more prominent watery discharge and scratchy sensation in viral conjunctivitis. It can be differentiated from bacterial conjunctivitis by its lack of purulent discharge. Allergic conjunctivitis is usually seen first in late childhood to early adulthood and often occurs in conjunction with symptoms of allergic rhinitis. The mainstay of treatment includes avoidance of allergens when possible and instilling topical mast cell stabilizers for daily prophylaxis against symptoms. Topical preparations with strictly antihistamine or vasoconstrictive effects may provide quicker symptom relief than daily mast stabilizers, but they hold a risk of causing rebound hyperemia and follicular conjunctivitis. Erythromycin ophthalmic ointment (A) is a topical antistaphylococcal antibiotic eye ointment that is applied to lid margins for conditions including blepharitis, hordeolum, and bacterial conjunctivitis. The patient above does not have symptoms of any condition necessitating an antistaphylococcal eye ointment. Prednisolone acetate ophthalmic suspension (C) is a topical steroid preparation indicated for inflammatory conditions of the anterior eye, however it poses the risk of causing cataracts and glaucoma over extended periods of use. Sulfacetamide ophthalmic solution (D) is a topical sulfonamide antibiotic appropriate in bacterial conjunctivitis, not allergic conjunctivitis.

Question: What diagnostic test is most sensitive in detecting valvular vegetations?

One Step Further Question: What diagnostic test is most sensitive in detecting valvular vegetations? Answer: Transesophageal echocardiogram (TEE). Rapid Review Endocarditis Patient will be complaining of fever, rash, cough and myalgias PE will show Fever, Roth spots, Osler nodes, Murmur, Janeway lesions, Anemia, Nailbed hemorrhages, Emboli (FROM JANE) Diagnosis is made by echocardiography and Duke's criteria Most commonly caused by: IVDA: S. aureus, tricuspid Native valve: Streptococci, mitral Treatment is antibiotics Comments: GI malignancy: S. bovis

Elderly, dizzy upon standing. No dizziness after prolonged standing and no dizziness with sitting or being supine. No chest pain, palpitations, or dyspnea. He denies associated chest pain, palpitations, or dyspnea. First test? Chest radiograph. Head-up tilt-table testing. Orthostatic vital signs. Transesophageal echocardiography.

Orthostatic vital signs. Orthostatic hypotension is due to an inadequate physiologic response to postural changes. This condition mostly exists in the elderly population. It is estimated that nearly 25% of syncopal admissions to the ED are due to orthostatic hypotension. Symptoms of orthostatic hypotension include dizziness, weakness, fatigue, light-headedness, headache or syncope which occur after standing. Primary causes include frailty, dehydration, poor cardiac output or autonomic instability. However, there are a multitude of underlying conditions that can cause secondary orthostasis. These include anemia, hemorrhage, cardiac dysfunction, venous insufficiency, endocrine dysfunction (hypothyroidism, hypoaldosteronism, adrenal insufficiency, diabetes insipidus, hypokalemia) and neurologic dysfunction (autonomic neuropathy, vitamin B12 deficiency). If suspected, orthostatic vital signs must be obtained as follows: BP and heart rate must be measured in the supine position, then repeated after 3 minutes of standing. If normal, but orthostasis is still suspected, then the patient should be sent for head-up tilt-table testing. Head-up tilt-table testing (B) can help confirm a diagnosis of suspected orthostatic hypotension when orthostatic vital signs are nondiagnostic. It should be done after orthostatic vital signs are checked. A chest X-ray (A) and echocardiogram (D) may be used during the evaluation of orthostasis, but they would not be ordered initially. It is important first to consider orthostatic hypotension as the diagnosis, then once confirmed, begin evaluation of other causes with more specific testing.

What treatment should be administered to patients with thyrotoxic periodic paralysis?

Potassium supplementation and beta blockers. On slightly related note, hypokalemia can be from diuretics, diarrhea, or vomiting. ECG shows U-waves, T-wave flattening, S.T. depressions, and QT prolongation. Check and replete magnesium.

Boy, 4. Adopted from Korea, arrived one week ago. Lived in orphanage since birth. Normal examination in Korea. Can copy a circle and draw a person with 2 body parts. Can walk up stairs with alternating feet. Shy, but interacts well with adoptive parents. Exam shows BMI 8th percentile. Dental caries. Which is true? Kids from South Korea should be screened for Chagas. Routine screening for HBV infection is recommended for internationally-adopted kids upon arrival. Routine screening for malaria is recommended for internationally adopted kids. Routine screening of internationally adopted kids for intestinal parasites is not needed upon arrival.

Routine screening for HBV infection is recommended for internationally-adopted kids upon arrival. Each year, thousands of children immigrate to the United States through international adoption. Internationally adopted children are at increased risk for common infectious diseases such as tuberculosis, intestinal parasites, skin infections, and infestations because they come from resource-poor countries where these conditions are common. The United States State Department requires that internationally adopted children undergo a medical examination in their birth country by a United States-designated physician before admission into the United States. Reevaluation for these conditions and evaluation for other illnesses and infections should be performed after arrival to the adoptive country. The American Academy of Pediatrics (AAP) and the Centers for Disease Control and Prevention (CDC) recommend routine screening for hepatitis B virus (HBV) infection for internationally adopted children upon arrival, regardless of results provided in the country of origin. Screening is necessary to identify asymptomatic carriers, who may nonetheless transmit HBV to household contacts and caregivers. Early identification permits appropriate management of the adopted child and preventive measures for contacts. Screening consists of assays for HBV surface antigen (HBsAg), HBV surface antibody (anti-HBs), and HBV core antibody (anti-HBc). Children who are adopted from South Korea should be screened for Chagas disease (A) is false. Screening for Chagas disease is done for patients from endemic areas such as Mexico, Central America, and South America. Routine screening for malaria is recommended for internationally adopted children (C) is wrong because routine screening for malaria is not recommended. Routine screening of internationally adopted children for intestinal parasites is not needed upon arrival (D) is false because routine screening for intestinal parasites should be done upon arrival. Screening involves obtaining three stool specimens to be examined for ova and parasites.

What is the most common risk factor for bacterial vaginosis?

Sexual activity. Rapid Review Bacterial Vaginosis: Patient will be complaining of malodorous vaginal discharge PE will show thin, gray/white discharge Labs will show pH > 4.5, clue cells Diagnosis is made by KOH smear → fishy odor, whiff test, Amsel criteria Most commonly caused by Gardnerella vaginalis Treatment is metronidazole

Woman, 27. Recently returned from camping in Florida. Now fever, headache, myalgias, shaking chills. A 27-year-old woman presents with fever. Elevated transaminases and thrombocytopenia. Which antibiotic? Ciprofloxacin. Erythromycin. Tetracycline. Trimethoprim-sulfamethoxazole.

Tetracycline. This patient's symptoms are consistent with ehrlichiosis. There are several species of Ehrlichia that cause infection in the United States and the species depends on the geographic location. The disease is spread to humans through a tick vector requiring a bite. On average, symptoms begin nine days after discovery of the tick bite. As opposed to other tick-borne illnesses like Lyme, ehrlichiosis is characterized by the abrupt onset of fever, headache, myalgias and shaking chills. Vomiting and diarrhea occur less frequently and up to 1/3 of patients develop a rash. Leukopenia, thrombocytopenia, and elevated liver enzymes occur in 50 to 90% of patients. Tetracycline and doxycycline are the antibiotics of choice and curative when administered for 7 to 14 days. Rifampin is an acceptable alternative. If patients do not improve after six to seven days, an alternative diagnosis should be considered. The diagnosis is made based on the clinical features and exposure, but is confirmed through several laboratory testing options: 1) antibody titers; 2) PCR assay for organism-specific DNA; 3) Identification of morulae in leukocytes; 4) Immunostaining on biopsy; 5) Culture. Ciprofloxacin (A), erythromycin (B) and trimethoprim-sulfamethoxazole (D) are not effective against Ehrlichia.

Girl, 6. Here with a limp. Parents noted she was having difficulty walking for two days. No trauma or fever, but she had a cold last week. She appears well, no fever here. Left hip is held in flexion and abduction. Labs show normal ESR, CRP, and white count. Ultrasound shows hip effusion. Diagnosis? Myositis. Septic arthritis. Stress fracture. Transient (toxic) synovitis.

Transient (toxic) synovitis. The girl has clinical and laboratory findings most consistent with transient synovitis. Toxic synovitis is a reactive arthritis that affects the hip. It is one of the most common causes of hip pain in children. It can manifest in all age groups with mean onset at six years of age. The majority of affected patients have an upper respiratory illness one to two weeks before the onset of symptoms. The symptoms usually develop acutely with pain in the groin, anterior thigh, knee or hip and children walk with a painful and limping gait. If a significant effusion is present, the hip is held flexed, abducted, or laterally rotated. Affected patients are often afebrile or have a low-grade fever. The diagnosis is made clinically and laboratory and radiographic tests are performed to rule out serious conditions. The ESR, CRP, and WBC are relatively normal. The radiographs of the pelvis when obtained are usually found to be normal. Ultrasonography of the hip may show joint effusion. Treatment is supportive that includes activity limitation and relief of weight bearing until the pain subsides. Myositis (A) may present in a child refusing to walk due to muscle pain. On examination, the calves are exquisitely tender, which was not evident in the girl in the vignette. Septic arthritis (B) can present with a limp but the pain is more severe and patients are more systemically ill with fever compared to toxic synovitis. Stress fracture (C) is considered in children who have history of new or increased sports activity, which is not present in the vignette.

Diastolic, rumbling. Bell over left sternal border at 4th intercostal space. Louder with inspiration. Which one? Aortic regurgitation. Aortic stenosis. Tricuspid regurgitation. Tricuspid stenosis.

Tricuspid stenosis. Tricuspid stenosis is a murmur that is heard best with the bell over the left sternal border at the fourth intercostal space. It can be caused by myxomatous degeneration, rheumatic heart disease, congenital malformations and endocardial fibroelastosis. It rarely occurs alone, and almost always occurs with mitral stenosis. It is usually described as a diastolic rumble, which is louder than mitral stenosis during inspiration. There may be an associated thrill. Overall, it is a relatively rare murmur, and when it is an isolated finding, it usually does not require treatment. However, as it commonly occurs with mitral stenosis, surgical valve replacement or balloon valvuloplasty may be necessary. Murmurs of the aortic valve (A and B) are most intense at the right, second intercostal space. Aortic regurgitation is a type of diastolic murmur but has no association wth Carvallo's sign. It is, however, associated with de Musset's sign (head nodding in time with the heart beat) and Quincke's sign (pulsation of the capillary bed in the nail). Tricuspid regurgitation (C) is a systolic, not diastolic, murmur.

A 23-year-old man with history of intravenous heroin abuse presents with fever of 38.5 and mild cough. Heart rate 133 and B.P. is 114 over 72. Ill-appearing, with diastolic murmur. Chest radiograph without pneumonia. Hospitalized two weeks ago for arm cellulitis. Which of the following antibiotic regimens is appropriate for this patient? Ceftriaxone and azithromycin. Nafcillin and vancomycin. Pencillin and nafcillin. Vancomycin and gentamicin.

Vancomycin and gentamicin. The combination of intravenous drug abuse, fever, and murmur is highly suspicious for infective endocarditis. The most common causative organism in this population is Staphylococcus aureus. The initial antibiotic regimen selected should reflect the susceptibilities of the suspected organism, the acuteness of the presentation, and local resistance patterns. Given the prevalence of methicillin-resistant Staphylococcus aureus (MRSA) in both the community and hospital setting, vancomycin, in addition to gentamicin (for its synergistic effects and coverage of gram-negative organisms), should be used for IV drug users with native valves. Ceftriaxone and azithromycin (A) would be an appropriate antibiotic choice if the patient had community-acquired pneumonia. But they will not provide sufficient coverage for suspected infective endocarditis. Nafcillin (B) is an excellent antibiotic to treat staph and strep infections. However, it is not effective for MRSA. This regimen also has poor gram-negative coverage (patients who abuse IV drugs are at risk for gram-negative endocarditis). Penicillin and nafcillin (C) are both effective for staph and strep species but ineffective against MRSA and do not cover gram-negative organisms well enough.


Related study sets

4.0 Project Integration Management (Multiple Choice)

View Set

DNA structure and function (a) 🦠

View Set

Chapter 7: Individual & Group Decision Making - How Managers Make Things Happen

View Set

Ch. 11: Healthy Eating for Healthy Babies

View Set

SERIES 6: Section 6 ... Suitability and Risk

View Set